2-body problem - conservation of angular momentum

In summary: If you consider extended bodies then angular momentum is conserved because the angular momentum of the whole is the sum of the angular momenta of the parts. The implicit assumption is that the interactions between extended bodies are accurately described as a simple linear superposition of the forces between all of the pairs of interacting parts.
  • #36
DrStupid said:
The other is at least as falsifiable as conservation of angular momentum. And of what avail is a predictable assumption if you have no idea if the prediction is correct or not? (And you have no idea if the assumption lacks proper justification.)

I still do not agree with your claim, that the 3rd law conserves angular momentum if you just assume the additional restriction of the direction of forces. It is your assumption that, together with the 3rd law, conserves angular momentum. Such a restriction must be justified (e.g. by experimental observations). If you just assume it than the resulting conservation of momentum is an assumption as well.
I am back to being baffled with what you are arguing about.
 
  • Like
Likes vanhees71
Physics news on Phys.org
  • #37
DrStupid said:
Angular momentum actually is conserved due to the central nature of the force. Forces have a build in conservation of linear momentum (thanks to Newton's 3rd law) but it is not obvious that they also conserve angular momentum. There must be an additional restriction that is not included into the definition of force. Why shouldn't it be mentioned in a book?

Angular momentum is conserved due a system being isolated from external torques. The central nature of any internal forces is irrelevant and angular momentum would be conserved if the internal forces were not central as long as the system was isolated from external torques
 
  • #38
dyn said:
I am back to being baffled with what you are arguing about.

As I wrote above: I do not agrere that the 3rd law conserves angular momentum just beause you make an arbitrary assumption.
 
  • #39
dyn said:
Angular momentum is conserved due a system being isolated from external torques.

That's obvious. We do not need to discuss about that. The question is if forces comply with conservation of angular momentum. They don't do that by definition (in contrast to conservation of linear momentum). There must be additional restrictions. That's where the central nature of internal forces comes into play.
 
  • #40
DrStupid said:
That's obvious. We do not need to discuss about that. The question is if forces comply with conservation of angular momentum. They don't do that by definition (in contrast to conservation of linear momentum). There must be additional restrictions. That's where the central nature of internal forces comes into play.

I'm shooting in the dark here, but sometimes in the cases where the forces in question are not central, don't you also need to worry about a bunch of other things like the momenta of fields? Self forces seemingly "violate" Newton III but apparently everything turns out okay if you factor in the momentum of the field. Is that relevant here?
 
  • #41
etotheipi said:
I'm shooting in the dark here, but sometimes in the cases where the forces in question are not central, don't you also need to worry about a bunch of other things like the momenta of fields?

Yes, if the forces are not central, momentum must be conserved otherwise and the moment of the field is one posibility. Lorentz force is an example where momentum is primary exchanged with a field and where the force doesn't act in the direction from the source of the field (for the case it has a clear location) to the affected particle. Something like that exceeds the original definition of forces which is limited to interactions between bodies.
 
  • Like
Likes etotheipi

Similar threads

Replies
5
Views
1K
Replies
3
Views
1K
Replies
1
Views
332
Replies
9
Views
1K
Replies
4
Views
1K
Replies
9
Views
3K
Replies
36
Views
14K
Replies
36
Views
3K
  • Mechanics
Replies
9
Views
4K
Back
Top